Тёмный

Powers of 2 (extra) - Numberphile 

Numberphile2
Подписаться 254 тыс.
Просмотров 32 тыс.
50% 1

A bit extra from the main video, which is at: • Problems with Powers o...
More links & stuff in full description below ↓↓↓
Neil Sloane is founder of the On-Line Encyclopedia of Integer Sequences: oeis.org
More of our videos with Neil Sloane: bit.ly/Sloane_Numberphile
NUMBERPHILE
Website: www.numberphile.com/
Numberphile on Facebook: / numberphile
Numberphile tweets: / numberphile
Numberphile is supported by the Mathematical Sciences Research Institute (MSRI): bit.ly/MSRINumberphile
Videos by Brady Haran
Support us on Patreon: / numberphile
Brady's videos subreddit: / bradyharan
A run-down of Brady's channels: www.bradyharan.com
Sign up for (occasional) emails: eepurl.com/YdjL9

Опубликовано:

 

14 ноя 2022

Поделиться:

Ссылка:

Скачать:

Готовим ссылку...

Добавить в:

Мой плейлист
Посмотреть позже
Комментарии : 39   
@carstenmeyer7786
@carstenmeyer7786 Год назад
To prove loops with four vertices cannot exist, consider such a loop with vertices *a1, a2, a3, a4:* *a1 -- a2 -- a3 -- a4 -- a1 // a4 connects back to a1* Assume *ak* stand for distinct integers, such that two neighboring *ak* sum to a power of two. We may assume *a3 > a1* and *a4 > a2* (if not, shift by two and/or change orientation): *a3 - a1 = (a2 + a3) - (a1 + a2) = 2^{m23} - 2^{m12} (1)* *a3 - a1 = (a4 + a3) - (a1 + a4) = 2^{m43} - 2^{m14} (2)* As they both represent *a3 - a1,* we have "(1) = (2)" and simplify to *2^{m43} + 2^{m12} = 2^{m14} + 2^{m23}* Notice *m43 > m12* (via *a4 > a2* and *a3 > a1* ), so the LHS is a binary representation of an integer with exactly two ones. But then, so is the RHS -- thus *m14 ≠ m23.* By uniqueness of binary representation, one of *{m14; m23}* must equal *m43* -- a contradiction to either *a1 ≠ a3* or *a2 ≠ a4,* respectively.
@retrogiftsuk4812
@retrogiftsuk4812 Год назад
I'm not sure if this is a different proof or just another way of saying the same thing but ... If you are looking for a square A B C D (back to A) and AB is a power of 2 and BC is a power of 2 then CD and DA cannot both be powers of 2. It's easiest to prove with an example Imagine A = -1 B = 3 C = 5 the difference between A and C is 6. there is only 1 pair of power of 2 numbers that has a difference of 6 (2 and 8. This is self explanatory as the lower number is at most half of the higher number so the difference is at least half the higher number and so only 1 number can be the solution for the higher number [the next power of 2 larger than the difference} and so there can only be 1 pair of powers of 2 with that difference) but the difference between CD and DA is the same as the difference between C and A (in this example 6) so the only pair of powers of 2 that CD and DA can be are the same as CB and BA making B and D the same number (which is not allowed) it doesn't matter what numbers A, B and C are, if AB and BC are powers of 2, for AD and DC to also be powers of 2, D and B have to be the same number (which is not allowed)
@carstenmeyer7786
@carstenmeyer7786 Год назад
@@retrogiftsuk4812 The two proofs are identical until the line where I compare both versions of *a3 - a1.* While you used the fact any (non-zero) difference of powers of two is unique to the ordered pair, I instead used the unique binary representation of natural numbers. The idea of both is essentially the same, though, I'd say. Never thought of the difference property, thank you for that!
@retrogiftsuk4812
@retrogiftsuk4812 Год назад
​@@carstenmeyer7786 thanks for the reply. As I said I wasn't sure if I was saying the same thing as you (I studied maths to A level a long time ago so I get lost very quickly in proofs written formally... hence my comment being written as more of a narrative)
@bandana_girl6507
@bandana_girl6507 Год назад
@@retrogiftsuk4812 That works as a proof against two loops of three joined on an edge. Ruling out loops of four (or larger) without the intermediate cross that my brain can't quite figure out on its own right now, but is the core of the proof for values up to 9 and the upper bound beyond that
@AdamZMouchnic
@AdamZMouchnic Год назад
I wish I watched this video yesterday evening. I watched the original, lost the interest a bit with the bounded solutions. During the evening walk I realized I can prove that a(4) < 6. I got hyped a little bit :) Anyway, my approach: In binary, a power of two is always one followed by arbitrary number of zeroes. So when you add two positive numbers, they have to be a sort of "opposite" of each other to get one followed by only zeroes. For example: 00001101011 01110010101 Leading zeroes can be ignored. There can also be trailing zeroes, but those just multiply the solution. The fun part is that you cannot add a third opposite. So, to create a set of three numbers, you have to pick a negative number. This was done before. But when you want to add the fourth number, it will either be positive and won't be able to form a subset of three, or it will be negative, and sum of two negative numbers is again negative.
@CarbonRollerCaco
@CarbonRollerCaco Год назад
You just showed me how to solve the whole thing-prove that a(4)=4. Fulfilling a(4)≥5 would require fulfilling a(3)=3, and since all solutions to a(3)=3 are, in binary form, -1Z, 11Z, 101Z where Z=any one string of zeroes as those are the only cases where one negative number can pair with both numbers of an opposite couple, the only remaining useful numbers are 1z001Z, -10O, 1Z, -100O, -11Z, o101Z, and o1011Z where z=any string of zeroes, O=any string of ones then zeroes the same length as Z and o=any string of ones. (Strings can be empty.) You can see for yourself that each one of those only pairs with one of the first three numbers. I'm certain proof that a(5)=6 and so on can be worked out from there. Of course, you could just use scientific notation to write any terminating number of any base and thus don't actually NEED to use other bases, but actually writing numbers IN other bases makes pattern visualization SO much easier and absolutely should NOT be slept on. Thank you for not only pointing the way for me to make child's play of a problem that stumped expert mathematicians, but further proving the value of thinking outside the box.
@Francesko901
@Francesko901 Год назад
That's funny! I just made a try to write down same idea... I just desperately didn't read your comment before writing my attempt as the comment.
@umanggada8684
@umanggada8684 Год назад
Could we see an update to this video explaining why it can't be a closed loop with 4 vertices?
@carstenmeyer7786
@carstenmeyer7786 Год назад
Consider a loop of four vertices *a1, a2, a3, a4:* *a1 -- a2 -- a3 -- a4 -- a1 // a4 connects back to a1* Assume *ak* stand for distinct integers, such that two neighboring *ak* sum to a power of two. We may assume *a3 > a1* and *a4 > a2* (if not, shift by two and/or change orientation): *a3 - a1 = (a2 + a3) - (a1 + a2) = 2^{m23} - 2^{m12} (1)* *a3 - a1 = (a4 + a3) - (a1 + a4) = 2^{m43} - 2^{m14} (2)* As they both represent *a3 - a1,* we have "(1) = (2)" and simplify to *2^{m43} + 2^{m12} = 2^{m14} + 2^{m23}* Notice *m43 > m12* (follows from assumption *a4 > a2* and *a3 > a1* ), so the LHS is a binary representation of an integer with exactly two ones. But then, so is the RHS -- thus *m14 ≠ m23.* By uniqueness of binary representation, one of *{m14; m23}* must equal *m43* -- a contradiction to either *a1 ≠ a3* or *a2 ≠ a4,* respectively.
@Muhahahahaz
@Muhahahahaz Год назад
@@carstenmeyer7786 thanks for the succinct proof! I was wondering how that worked… (Minor note: I think the last inequality should be a2 ≠ a4?)
@carstenmeyer7786
@carstenmeyer7786 Год назад
@@Muhahahahaz You're correct, of course. Thank you for pointing out that typo, it's fixed now!
@davidvegabravo1579
@davidvegabravo1579 Год назад
I REALLY LOVE THIS GUY
@nikoskonstantinou3681
@nikoskonstantinou3681 Год назад
That's amazing!
@AssemblyWizard
@AssemblyWizard Год назад
So a lower bound of n*log(n), and the 4-cycle thing gives us an upper bound of n*sqrt(n), whereas the total number of pairs is n² (all asymptotically up to a constant multiple). There's much leeway between n*log(n) and n*sqrt(n), but pretty cool progress so far
@Qermaq
@Qermaq Год назад
This is bananas. B-A-N-A-N-A-S.
@snowbie.
@snowbie. Год назад
cheeky unlisted video
@lyrimetacurl0
@lyrimetacurl0 Год назад
Damn
@BAbdulBaki
@BAbdulBaki Год назад
What happens if you require no duplicates of powers of 2? No duplicates of any number sums?
@SuperYoonHo
@SuperYoonHo Год назад
Hi neil!❤
@DaviddeKloet
@DaviddeKloet Год назад
This video showed up in my feed 11 hours ago, but it has comments from a month ago. What happened?
@visualgarden2718
@visualgarden2718 Год назад
Could someone help me here? Why is the generalized form of this n-1? Wouldnt it be n+1?
@Muhahahahaz
@Muhahahahaz Год назад
0:54 are you talking about this part? We want a list of n different integers. If we start with M - n and go by 2s, then we need to add n - 1 new numbers to the list. Thus the last number will be 2(n - 1) larger than M - n
@PeterBarnes2
@PeterBarnes2 Год назад
I wonder if graphs of solutions are planar.
@hhhguir
@hhhguir Год назад
They must be, since both forbidden minors for planarity (K_5 and K_3,3) contain 4-cycles, but a solution's graph cannot, as shown by M. S. Smith
@ogxj6
@ogxj6 Год назад
Hello everyone enjoying secret video
@domzi
@domzi Год назад
Interesting topic, thanks for that. But I cooment for something else: the video was released ome day ago, but it did not show up on my subscription list … strange. I wonder how many videos I missed because of this
@TheElectricCaveman
@TheElectricCaveman Год назад
I noticed the same. By some brief poking around, it looks like this video is unlisted altogether for some reason
@PhilBoswell
@PhilBoswell Год назад
If you're referring to this video, it's still unlisted, so it won't show up on anybody's notifications. As for the main video, I couldn't say, sorry: it's showing up for me.
@allovas
@allovas Год назад
It has no been made public yet
@AssemblyWizard
@AssemblyWizard Год назад
The video is unlisted, you can't see it without a direct link
@ogxj6
@ogxj6 Год назад
Congratulations you found a secret video.
@toxxicdutchie232
@toxxicdutchie232 Год назад
Ugh he said "no one knows" now its gonna haunt me to try and prove it.
@jeremykeens3905
@jeremykeens3905 Год назад
What the heck was that about!
@billr3053
@billr3053 Год назад
Exactly. Seems to have come out of nowhere. No context.
@synchronos1
@synchronos1 Год назад
@@billr3053 the context is given on the first line of the description. Watch the main video first and then watch this as an extra footage of the same topic. Most of the videos on Numberphile2 are some extra bits that didn't make it to the main video on Numberphile.
@Francesko901
@Francesko901 Год назад
I hope I found easy solution, why it can't work with 4 distinctive integers. The key is in closer look on how we solve it successfully for 3. 1) for successful solution for 3 you need one negative integer: sum of 2 negative integers can't be power of 2, obviously, for same reason we can't use one negative, zero and one positive, since sum of zero and negative can't be power of 2, obviously, and why you can't use 3 positive integers or 2 positive and zero I will show later. 2) then, if we need one negative integer for solution with 3, we definitely need two negative integers for solution with 4, but obviously sum of two negative integers can't be power of 2 That's the elegant part of proof, the rest is not as elegant... 3) the proof why we need three positive integers is based on fact that if you sum two DIFFERENT powers of 2, you don't receive another power of 2, since 2^n + 2^(n+1) < 2^(n+2), Then we instead focusing on summed integers focus firstly on their sums: we decompose all powers of 2 into all possible couples of different integers, then we easily find two powers of 2 sharing one soluting integer in their couples, i.e. we find in decomposition of one power of 2 a couple with one integer which we can find in a decomposing couple for another power of 2, BUT the other halves of these couples don't sum up to the power of two; if you find more elegant proof that three positive integers don't work as the solution, then it really improves the proof
@PiR2InTheUSA
@PiR2InTheUSA Год назад
What?? Please provide some context.
@SoleaGalilei
@SoleaGalilei Год назад
Perhaps you should watch the main video before the extra.
Далее
Problems with Powers of Two - Numberphile
10:57
Просмотров 313 тыс.
Crepe roll 🫶 #abirzkitchen #cooking
00:59
Просмотров 995 тыс.
Dungeon Numbers - Numberphile
13:48
Просмотров 337 тыс.
Top 10 Trading Rules - Jim Simons
2:26
Просмотров 3,8 тыс.
An Integration Conundrum - Numberphile
14:32
Просмотров 214 тыс.
Stones on an Infinite Chessboard - Numberphile
17:05
Просмотров 353 тыс.
The Foundation of Mathematics - Numberphile
15:11
Просмотров 75 тыс.
Dirac's belt trick, Topology,  and Spin ½ particles
59:43
The Brussels Choice - Numberphile
16:38
Просмотров 310 тыс.
Untouchable Numbers - Numberphile
8:09
Просмотров 123 тыс.